Gyroscope - I just don't get it.

  • Thread starter quasar987
  • Start date
  • Tags
    Gyroscope
In summary, a rotating gyroscope is able to resist the force of gravity and maintain its orientation due to its angular momentum. This momentum, which is perpendicular to the direction of gravity, keeps the gyroscope stable and prevents it from falling to the ground. When the gyroscope is at an angle, the downward force of gravity is increased, but the vertical component of angular momentum must be conserved, so the gyroscope continues to rotate about the vertical axis. This is in contrast to a non-rotating gyroscope, which will fall to the ground while gaining a small amount of angular momentum in the x-z plane due to the force of gravity. The spinning top's resistance to rotating is what allows it to maintain its orientation and appear to
  • #36
Eric Laithwaite was one of those most dangerous creatures: a career-crackpot who managed (I blame it on a shortage of university manpower after WWII) to get himself into a position of academic power. The gyroscope affair was only the tip of the iceberg. He also fancied that he was an expert on moths, and maintained a ludicrous theory (radio) for their communication even when the relevant pheromone (the true explanation) could be bought 'over-the-counter'. He did not even invent the linear induction motor, for which he is often credited. It was invented by Wheatstone. He also did not invent Maglev: an experimental levitated railway was already in operation when Laithwaite was a child. He did not develop a gyroscopic propulsion device; he got patents for several. This means nothing. The patent office is a joke, and let's any nonsense go by (they get their fees regardless). One of the leading free-energy (perpetual motion) promoters is a former patent-agent! I call Laithwaite dangerous because he has seemingly infected thousands of people with the idea that gyroscopes have anomalous properties and, more importantly, he has made acceptable the performance of sloppy experiments, and grandstanding for the benefit of journalists. His name should rightfully become a pejorative term among scientists; like Quisling is among politicians.
 
Physics news on Phys.org
  • #37
My question is:

which forces are present in the case of precessing gyroscope?
I have drawn a simple picture of the situation. The red force is the gravitational force. Is there a centripetal force -green? I think there must be since the mass is rotating. With the green force is easy to get why the mass is not falling. Or the green force is just fictional?
 

Attachments

  • gyro.jpg
    gyro.jpg
    7.4 KB · Views: 422
  • #38
What is included in the articles but not clearly stated in this thread is that the gyroscope "reacts" to a torque about 90 degrees out of phase. On an obect spinning about the vertical axis, a torque along the roll axis results in pitch reaction, and vice versa. This is how helicopters are controlled, roll and pitch torques at the rotor blades result in pitch and roll reactions respectively, so the control inputs are shifted in the "cyclic" 90 degrees out of phase so the control input stick corresponds to the anticipated reaction.

In the case of a gyroscope aligned so it's primary axis is horizontal, then a roll torque results in a yaw reaction, except that since one end of the gyroscope is fixed at a point, the yaw reaction is resisted. The result is that the gyroscope will react with both roll and yaw until it's rate of yaw matches the precession speed. If there is no dampening (a frictionless base), then the gyrosope rate of yaw, and direction of it's primary axis would cycle.

If the frame supporting the gyroscope were placed on a scale, then the scale would show a lower "weight" when the gygroscopes center of mass was accelerating downwards and a higher weight during upwards acceleration, as the gyroscope rate of yaw and primary axis direction cycles. If the initial condition is such that the rate of yaw is the same as the "precession" speed, the torques cancels and the rate of yaw and direction of primary axis remain constant (assuming no frictional losses), there would be no component of vertical acceleration and the weight shown on the scale would be constant and equal to the weight of gyroscope and supporting frame.
 
  • #39
This gentlemen, is exactly the point<

everything here revolves around centifugal force of an object that is balanced and spinning, now we know that this balanced spinning object has a center line, eg a center line that the object spins around,
Now if you introduce, every action has an opposite and equal action against it we can see how the the top maintains its centre point, and weight.

Introduce another balanced spinning object around the first object using the same center(plane), then you have a gyro that balances itself by offsetting one weight against the other in diff directions
 
  • #40
quasar987 said:
I don't understand why a rotating gyroscope whose rotation axis is inclined with respect to a vertical axis does not fall to the ground dues to gravity as fast as the non-rotating giroscope.

The answer to that has two stages. First the physics of gyroscopic precesssion.

This section from the http://en.wikipedia.org/wiki/Precession#Torque-induced" explains with a picture why gyroscopic precession occurs in the first place.

In the following post here on physics forums
https://www.physicsforums.com/showpost.php?p=2375054&postcount=3
I discuss the classroom demonstration of a spinning bicycle wheel, with horizontal axis, supported on only one axle end. Ultimately gyroscopic precession accounts for the chain of events that makes the gyroscope not topple over immediately.

Cleonis
 
Last edited by a moderator:
  • #41
To Cleonis : The question you've just answered has been asked in 2004, November.
The asker also understood the answers he got in the thread, clearing his initial doubt.
 

Similar threads

Replies
23
Views
972
  • Classical Physics
Replies
10
Views
1K
Replies
19
Views
1K
  • Classical Physics
Replies
1
Views
732
Replies
0
Views
510
  • Introductory Physics Homework Help
Replies
33
Views
954
  • Classical Physics
Replies
2
Views
951
  • Classical Physics
2
Replies
35
Views
3K
  • Mechanics
Replies
10
Views
1K
Replies
8
Views
2K
Back
Top